Lagrange's Theorem: Clarifying | | Meaning

  • Thread starter Thread starter Firepanda
  • Start date Start date
  • Tags Tags
    Theorem
Click For Summary
Lagrange's Theorem states that for a finite group G and its subgroup H, the order of H is a divisor of the order of G, expressed as |G| = m|H|, where m represents the number of cosets. The notation |G| indeed refers to the number of elements in the group G. In the case of the symmetric group S4, which has 24 elements, it is impossible to have a subgroup of order 5 because 24 is not a multiple of 5. The confusion arises from equating the number of elements with the concept of order, but in group theory, the order of a group is defined as the total count of its elements. Understanding this distinction clarifies why certain subgroup orders are not possible.
Firepanda
Messages
425
Reaction score
0
The definition I have is:

Leg G be a finite group and let H be a subgroup of G. Then the order of H is a factor of the order of G. More precisely, |G|=m|H| where m is the number of different cosets of H in G.

Can someone clarify what the | | means?

I thought it was how many elements are in a group, such as, the symmetric group of 4 (S4) has 4! elements, so |S4| = 24.

I have an example saying the S4 cannot have a subgroup of order 5 since |S4| = 24 which is not an exact multiple of 5.

But 24 is the number of elements in S4, not the order of the group.. So why are they saying the subgroup can't have order 5 because of the number of elements in S4? Surely we should be finding the order of S4 instead to se if there is a subgroup of order 5 in the group.

Thanks
 
Physics news on Phys.org
The order of a group G is the number of elements in the group G, which is denoted as |G|. I don't know what is confusing you.
 
Question: A clock's minute hand has length 4 and its hour hand has length 3. What is the distance between the tips at the moment when it is increasing most rapidly?(Putnam Exam Question) Answer: Making assumption that both the hands moves at constant angular velocities, the answer is ## \sqrt{7} .## But don't you think this assumption is somewhat doubtful and wrong?

Similar threads

  • · Replies 6 ·
Replies
6
Views
2K
  • · Replies 5 ·
Replies
5
Views
3K
  • · Replies 7 ·
Replies
7
Views
2K
  • · Replies 7 ·
Replies
7
Views
2K
  • · Replies 2 ·
Replies
2
Views
4K
Replies
2
Views
2K
  • · Replies 23 ·
Replies
23
Views
3K
  • · Replies 1 ·
Replies
1
Views
2K
  • · Replies 1 ·
Replies
1
Views
531
  • · Replies 2 ·
Replies
2
Views
4K